La ricerca ha trovato 145 risultati

da Higgs
27 apr 2024, 11:33
Forum: Problemi teorici
Argomento: 3 voltmetri e 3 resistenze
Risposte: 2
Visite : 56

Re: 3 voltmetri e 3 resistenze

Nessuno mi ha risposto comunque mi è riuscito trovando il risultato ufficiale di 12 V supponendo che i voltmetri abbiano, contrariamente a quello che avevo supposto, una resistenza finita pari a 3R e che quindi assorbano corrente! :D
da Higgs
24 apr 2024, 17:59
Forum: Problemi teorici
Argomento: 3 voltmetri e 3 resistenze
Risposte: 2
Visite : 56

3 voltmetri e 3 resistenze

Ho un problema tratto da Khalda la cui figura non riesco a riprodurre nel forum. Provo a descriverlo. Si tratta in DC di un circuito formato da tre maglie rettangolari contenenti ciascuna un voltmetro ed una resistenza. I voltmetri sono identici così come le resistenze R. Il primo lato verticale del...
da Higgs
10 apr 2024, 11:47
Forum: Problemi teorici
Argomento: Sovrapposizione di stati: teorica o vera?
Risposte: 3
Visite : 1588

Re: Sovrapposizione di stati: teorica o vera?

E' morto Higgs, provo una grande tristezza. Secondo me non si può osservare ciò che non esiste. Sennò la realtà sarebbe una creazione dell'osservatore. Come dire che l'osservatore esiste se viene osservato. Può essere una posizione filosofica idealista ma non scientifica sperimentale. E la scienza, ...
da Higgs
8 gen 2024, 19:24
Forum: Problemi teorici
Argomento: SNS n.5,2023
Risposte: 19
Visite : 4997

Re: SNS n.5,2023

Senza m abbiamo che il legame fra U e V=V_2 è l'equazione di continuità. U A^2= \sqrt{2gh}.a^2 da cui segue U_0=\sqrt{2gh}.\frac{a^2}{A^2} . Applicando il teorema di Bernoulli e considerando che A_serb>>a si ottiene V{\simeq}\sqrt{2gh} La presenza di m comporta un lavoro negativo effettuato su U da ...
da Higgs
6 gen 2024, 19:05
Forum: Problemi teorici
Argomento: SNS n.5,2023
Risposte: 19
Visite : 4997

Re: SNS n.5,2023

a) Ho capito finalmente da entrambi alcune cose importanti che ricapitolo per andare avanti sempre che siano corrette. Lo scopo di questo post è abbozzare la soluzione definitiva di a). In seguito affronterò b) perché devo digerire l'ultima esauriente trattazione di Tarapia a1) Scrivendo (Pigkappa) ...
da Higgs
5 gen 2024, 12:29
Forum: Problemi teorici
Argomento: SNS n.5,2023
Risposte: 19
Visite : 4997

Re: SNS n.5,2023

@ Tarapia Intanto desidero ringraziarti per la tua trattazione che sfiora questioni come la statistica su cui non sono preparato. Stamani ho rifatto i conti sul punto a) che mi farebbe realizzare un'incertezza percentuale su \alpha del 5,7% e quindi in linea con la richiesta del testo. Più difficile...
da Higgs
3 gen 2024, 19:29
Forum: Problemi teorici
Argomento: SNS n.5,2023
Risposte: 19
Visite : 4997

Re: SNS n.5,2023

a)Per la densità della sfera avrei trovato \rho=\rho_W.\frac{V_R - V_C}{V_R}= \frac {10^3kg/m^3[(500\pm15) 10^{-6}m^3 - (50\pm2)10^{-6}m^3]}{(500\pm15)10^{-6}m^3} = 10^3kg/m^3 \frac{450\pm17}{500\pm15}= 900[(1\pm(1\pm6%)]kg/m^3 . Pertanto mi risulterebbe S-P= (\rho_W - \rho)V_R g N= 100(1\pm 6%).500...
da Higgs
2 gen 2024, 19:30
Forum: Problemi teorici
Argomento: SNS n.5,2023
Risposte: 19
Visite : 4997

Re: SNS n.5,2023

a) Ho capito finalmente da entrambi alcune cose importanti che ricapitolo per andare avanti sicuro della loro correttezza. Lo scopo di questo post è abbozzare la soluzione definitiva di a). In seguito affronterò b) perchè devo digerire alcuni punti dell'ultima esauriente trattazione di Tarapia a1) S...
da Higgs
30 dic 2023, 19:01
Forum: Problemi teorici
Argomento: SNS n.5,2023
Risposte: 19
Visite : 4997

Re: SNS n.5,2023

In attesa della risposta ad a) vorrei provare anche con b). La differenza fra i due casi richiesti dal testo consiste nel fatto che in un caso (presenza di m) questo esercita sul fluido per la relatività del moto la forza opposta alla velocità del fluido e dopo c'é la differenza di portata A/a mentr...
da Higgs
28 dic 2023, 12:57
Forum: Problemi teorici
Argomento: SNS n.5,2023
Risposte: 19
Visite : 4997

Re: SNS n.5,2023

@Pigkappa 1) Castroneria mia su \rho . Il bello è che pensavo a una cosa e ne ho scritta un'altra. Comunque sono d'accordo: a me verrebbe \rho= 900\pm63 gr/cm^3 2) Per quanto riguarda invece la soluzione di V(t) trovo che la mia sia più immediata e veloce nel determinare la velocità limite. Infatti ...